LSAT and Law School Admissions Forum

Get expert LSAT preparation and law school admissions advice from PowerScore Test Preparation.

 Administrator
PowerScore Staff
  • PowerScore Staff
  • Posts: 8916
  • Joined: Feb 02, 2011
|
#40585
Complete Question Explanation
(The complete setup for this game can be found here: lsat/viewtopic.php?t=11817)

The correct answer choice is (A)

In Global, could Be True questions, look to two sources for assistance: Not Laws and rules to eliminate incorrect answers, and prior work to possibly prove a correct answer.

If you look at prior work, in question #21, in Solution #1, O is fifth and given at G. Thus, answer choice (A) could be true and is therefore the correct answer.

If you did not see this connection, you can still eliminate several incorrect answers quickly via the rules and Not Laws:

Answer choice (B) can be eliminated because the fifth rule stipulates that P is given at G.

Answer choice (C) can be eliminated because it leaves no room for O (if R is fifth and given at H, the other H is still fourth. From the fourth rule, it would have to be S. That would then force O to be fifth, which is impossible in this answer since it stipulates that R is fifth).

And from the Not Laws, the fifth lecture cannot be S or T, and thus answer choices (D) and (E) can immediately be eliminated.

Thus, regardless of the path used, answer choice (A) could be true and is the correct answer.

Get the most out of your LSAT Prep Plus subscription.

Analyze and track your performance with our Testing and Analytics Package.